Solving Hyperbolic Functions: cothx - \frac{1}{x}

Click For Summary
The discussion revolves around proving the relationship between xcothx and cothx - 1/x, with the initial claim being that they are equal. However, upon testing with x = 1, it becomes clear that the two expressions do not match, leading to the conclusion that the original assertion is incorrect. Participants emphasize the importance of verifying steps in mathematical proofs, particularly when using functions like coth. The conversation also touches on the application of the quotient rule in calculus to simplify expressions. Ultimately, the thread highlights the necessity of careful mathematical reasoning and validation of claims.
Brewer
Messages
203
Reaction score
0
[SOLVED] Hyperbolic functions

As part of a long winded "show that" question I've ended up at the point where I have xcothx and I want to show that this is equal to cothx - \frac{1}{x} only I have no ideas how to get there. I can't see any reason why this should be so, but I'm pretty confident that I'm correct so far (in fact I know I am!).

Obviously I could take the "magic step" when doing this kind of question (in that I could just write the final answer down, and hope that I'm close enough for this step to be intuitive) but I'd quite like to know the step to take.

Thanks in advance guys.
 
Physics news on Phys.org
erm … when x = 1, xcothx = coth1, but cothx - 1/x = coth1 - 1 … so they're not equal. :frown:

How did you get to that position?
 
Well, you aren't going to be able to prove that because it pretty obviously isn't true. In particular, you would be proving, taking x= 1, that coth(1)= coth(1)- 1 which can't be true.
 
Meh, so the guy I checked my working with up until now is wrong.

So what I did is
<br /> U=-\frac{d(lnZ)}{d\beta}
=-\frac{1}{z}\frac{dZ}{d\beta}
=\frac{-\beta \mu B}{sinh(\beta \mu B)}(\mu Bcosh(\beta \mu B)
=-\frac{\mu ^2 B^2 \beta cosh(\beta \mu B)}{sinh(\beta \mu B)
=-\mu B(\beta \mu Bcoth(\beta \mu B)where Z = \frac{sinh(\beta \mu B)}{\beta \mu B}

Are there problems with what I've done then?
 
Last edited:
I've tried to correct the latex, but if it hasnt come out properly then the line that's gone wrong is the unfactorised form of the final line
 
Solved. Helps when you remember your quotient rule
 
Question: A clock's minute hand has length 4 and its hour hand has length 3. What is the distance between the tips at the moment when it is increasing most rapidly?(Putnam Exam Question) Answer: Making assumption that both the hands moves at constant angular velocities, the answer is ## \sqrt{7} .## But don't you think this assumption is somewhat doubtful and wrong?

Similar threads

  • · Replies 1 ·
Replies
1
Views
1K
  • · Replies 5 ·
Replies
5
Views
1K
  • · Replies 4 ·
Replies
4
Views
1K
  • · Replies 4 ·
Replies
4
Views
1K
  • · Replies 7 ·
Replies
7
Views
1K
  • · Replies 5 ·
Replies
5
Views
2K
  • · Replies 6 ·
Replies
6
Views
5K
  • · Replies 7 ·
Replies
7
Views
3K
  • · Replies 4 ·
Replies
4
Views
2K
  • · Replies 5 ·
Replies
5
Views
1K